1

I am very frustrated with the fact that the E and B field of an EM wave are in phase in the far-field case.

Before I get into this question I am letting you know that I already read this link related to my question, but it didn't really help me so much.

First, I know that an EM wave propagates as shown in the GIF below.

enter image description here

But, how are E and B field in phase? According to Maxwell's equation #3 and 4, we know that the change of E field induces B field, and vice versa. I also attached these equations below. I deleted the J term since we are talking about the far-filed analysis which lets us ignore the B field induced by the current flowing, J, in the wire.

enter image description here

Let's assume that the wave propagates along the x-axis. According to Maxwell, when there is a maximum E filed occurs (dE(t)/dt = 0) at a given point, x0, a minimum B field (B=0) should occur at the point. This should result in 90° phase difference between E and B field.

I know that we can calculate and prove that the EM wave propagates at a speed of light from Maxwell's equation #3 and 4 as shown below (source).

enter image description here

But the problem is that I don't know how to solve partial derivative equations (PDEs), and also I want some visualizations to understand this phenomenon better.

Is there any easy way to visualize why it is like this without solving PDEs?

Thank you.

  • "*Let's assume that the wave propagates along the x-axis. According to Maxwell, when there is a maximum E filed occurs (dE(t)/dt = 0) at a given point, x0, a minimum B field (B=0) should occur at the point. This should result in 90° phase difference between E and B field.*" A moment when both waveforms have a slope of zero at the same time doesn't sound like 90 degrees out of phase to me. If anything that's either in phase or 180 degrees out of phase. – DKNguyen Mar 23 '20 at 19:44
  • https://physics.stackexchange.com/questions/181277/do-the-electric-and-magnetic-components-of-an-electromagnetic-wave-really-genera?rq=1 – Sredni Vashtar Mar 24 '20 at 01:00
  • Hi, thank you for your answer to my question. In my example, when the slope of E is 0 and the magnitude of B is 0 (not the slope of it). The slope of B would be higher or lower than 0 which means that E and B are out of phase, but they have to be in phase according to Maxwell... – Young Soung Park Mar 25 '20 at 05:25
  • Sredni Vashtar, thank you for your link. I looked at it and it helped me understanding my issue a bit better. Though, I still don't understand fully. – Young Soung Park Mar 25 '20 at 05:36

2 Answers2

3

The impedance of free space is not a complex impedance quantity; it is resistive with a value of \$120\pi\$ or about 377 ohms. This means that the magnitude ratio of E to B is 377:1 and, each are linked in phase to one another due to that non-complex resistance. If you expect a full derivation of Maxwell's equations on this site, you are expecting too much - try stack exchange physics.

Andy aka
  • 434,556
  • 28
  • 351
  • 777
0

someone wise said "Whether Efield or Hfield, its all caused by electric charges"

When you have changing electric fields, the (very small time delays along a wire) might result in what philosophers have chosen to label magnetically-induced voltages and currents.

analogsystemsrf
  • 33,703
  • 2
  • 18
  • 46